ABFM ITE 2019 Flashcards
2019.1)
A 42-year-old female presents for follow-up after being treated for recurrent respiratory problems at an urgent care facility. She is feeling a little better after a short course of oral prednisone and use of an albuterol (Proventil, Ventolin) inhaler. She has had a gradual increase in shortness of breath, a chronic cough, and a decrease in her usual activity level over the past year. She has brought a copy of a recent chest radiograph report for your review that describes panlobular basal emphysema. She does not have a history of smoking, secondhand smoke exposure, or occupational exposures. Spirometry in the office reveals an FEV1/FVC ratio of 0.67 with no change after bronchodilator administration. Which one of the following underlying conditions is the most likely cause for this clinical presentation?
A) α1-Antitrypsin deficiency
B) Bronchiectasis
C) Diffuse panbronchiolitis
D) Interstitial lung disease
E) Left heart failure
A) α1-Antitrypsin deficiency (REVIEW: 2018.126 )
This patient presents with symptoms of chronic obstructive lung disease, and spirometry confirms airflow limitation or obstruction with an FEV1/FVC <0.7. Her age, the lack of tobacco smoke or occupational exposures, and the chest radiograph findings are typical of 1-antitrypsin deficiency. While left heart failure, interstitial lung disease, bronchiectasis, and diffuse panbronchiolitis are all causes of chronic cough, they are not necessarily associated with the development of COPD and these spirometry findings. Furthermore, the radiologic findings in this patient are not consistent with these conditions.
- Left heart failure would present with pulmonary edema on a chest radiograph and volume restriction on pulmonary function testing.
- Bronchiectasis would present with bronchial dilation and bronchial wall thickening on a chest radiograph.
- Interstitial lung disease would present with reticular or increased interstitial markings.
- Diffuse panbronchiolitis would present with diffuse small centrilobular nodular opacities along with hyperinflation.
2019.2)
An otherwise healthy 57-year-old male presents with mild fatigue, decreased libido, and erectile dysfunction. A subsequent evaluation of serum testosterone reveals hypogonadism.
Which one of the following would you recommend at this time?
A) No further diagnostic testing
B) A prolactin level
C) A serum iron level and total iron binding capacity
D) FSH and LH levels
E) Karyotyping
D) FSH and LH levels
In men who are diagnosed with hypogonadism with symptoms of testosterone deficiency and unequivocally and consistently low serum testosterone concentrations, further evaluation with FSH and LH levels is advised as the initial workup to distinguish between primary and secondary hypogonadism.
- If secondary hypogonadism is indicated by low or inappropriately normal FSH and LH levels, prolactin and serum iron levels and measurement of total iron binding capacity are recommended to determine secondary causes of hypogonadism, with possible further evaluation to include other pituitary hormone levels and MRI of the pituitary.
- If primary hypogonadism is found, karyotyping may be indicated for Klinefelter’s syndrome.
2019.3)
A 4-year-old female is brought to your office because of a history of constipation over the past several months. Her mother reports that the child has 1–2 bowel movements per week composed of small lumps of hard stool. She strains to have the bowel movements, and they are painful. The child eats normally like her two siblings.
Which one of the following would be most effective at this time?
A) Daily fiber supplements
B) Lactulose
C) Magnesium hydroxide (Milk of Magnesia)
D) Polyethylene glycol (MiraLAX)
E) Senna
D) Polyethylene glycol (MiraLAX) (REVIEW: 2018.52)
This patient presents with symptoms compatible with functional constipation. Daily use of polyethylene glycol (PEG) solution has been found to be more effective than lactulose, senna, or magnesium hydroxide in head-to-head studies.
- Evidence does not support the use of fiber supplements in the treatment of functional constipation.
No adverse effects were reported with PEG therapy at any dosing regimen. Low-dose regimens of PEG are 0.3 g/kg/day and high-dose regimens are up to 1.0–1.5 g/kg/day.
2019.4)
A 30-year-old female presents with a 5-day history of subjective fever and malaise. She does not have a thermometer at home but has felt alternately warm and chilled. She has felt generally unwell and is sleeping more than usual. She has had a decreased appetite but has been drinking fluids without difficulty. She does not have a runny nose, cough, headache, abdominal pain, vomiting, diarrhea, joint pain, rash, or pain with urination. Her medical history includes substance use disorder and she takes buprenorphine/naloxone (Suboxone). She smokes one pack of cigarettes daily, has 0–2 alcoholic drinks daily, and began using intravenous heroin again 1 week ago.
An examination reveals a blood pressure of 112/68 mm Hg, a pulse rate of 88 beats/min, a respiratory rate of 16/min, a temperature of 38.9°C (102.0°F), and an oxygen saturation of 95% on room air. The patient appears fatigued and uncomfortable but nontoxic. Her heart has a regular rate and rhythm with no murmur. Her lungs are clear to auscultation bilaterally and her abdomen is soft and nontender. There is no swelling or redness in the extremities and a skin examination reveals no rashes or lesions.
Which one of the following would be most important at this point?
A) A viral swab
B) An antinuclear antibody level
C) Blood cultures
D) An erythrocyte sedimentation rate
E) A chest radiograph
C) Blood cultures
A patient who uses intravenous drugs and has a fever without a clear source must be evaluated for infectious endocarditis (IE).
- The first step in this evaluation is to obtain blood cultures. Although this patient might have a less serious condition, it is critical to evaluate for bacteremia in this situation.
- If the concern for IE is high, blood cultures should be obtained and antibiotics may be started while waiting for results and arranging for urgent echocardiography.
IE in people who inject drugs is more likely to be right-sided, specifically involving the tricuspid valve. Right-sided IE is less frequently associated with systemic findings of endocarditis such as Janeway lesions or Roth spots. Patients often do not have a heart murmur.
2019.5)
During a newborn examination you note a foot deformity, with the front half of the foot turned inward. Applying gentle pressure to the forefoot while holding the heel steady brings the heel and forefoot into alignment.
Which one of the following would you recommend?
A) Observation only
B) Adjustable shoes
C) Serial casting
D) Surgical correction
A) Observation only
This patient has flexible metatarsus adductus, the most common congenital foot deformity. Flexible metatarsus adductus usually resolves spontaneously by 1 year of age and does not require treatment.
- Rigid metatarsus adductus should be treated with serial casting. Using adjustable shoes is an alternative that is less expensive than serial casting for motivated parents with children who are not yet walking.
- Surgical correction should be reserved for older children who are already walking or for those with persistent symptomatic metatarsus adductus that is resistant to casting.
2019.6)
A 35-year-old female comes to your office for evaluation of a tremor. During the interview you note jerking movements first in one hand and then the other, but when the patient is distracted the symptom resolves. Aside from the intermittent tremor the neurologic examination is unremarkable. She does not drink caffeinated beverages and takes no medications.
Which one of the following is the most likely diagnosis?
A) Parkinson’s disease
B) Cerebellar tremor
C) Essential tremor
D) Physiologic tremor
E) Psychogenic tremor
E) Psychogenic tremor
Psychogenic tremor is characterized by an
- Abrupt onset,
- Spontaneous remission,
- Changing characteristics, and
- Extinction with distraction.
Cerebellar tremor is an intention tremor with ipsilateral involvement on the side of the lesion. Neurologic testing will reveal past-pointing on finger-to-nose testing. CT or MRI of the head is the diagnostic test of choice.
Parkinsonian tremor is noted at rest, is asymmetric, and decreases with voluntary movement. Bradykinesia, rigidity, and postural instability are generally noted. For atypical presentations a single-photon emission CT or positron emission tomography may help with the diagnosis. One of the treatment options is carbidopa/levodopa.
Patients who have essential tremor have symmetric, fine tremors that may involve the hands, wrists, head, voice, or lower extremities. This may improve with ingestion of small amounts of alcohol. There is no specific diagnostic test but the tremor is treated with propranolol or primidone.
Enhanced physiologic tremor is a postural tremor of low amplitude exacerbated by medication. There is usually a history of caffeine use or anxiety.
2019.7)
A patient with moderately severe Alzheimer’s disease has been taking quetiapine (Seroquel), 50 mg daily at bedtime, to manage behavioral symptoms related to the dementia. The patient’s symptoms have been stable on the quetiapine for 6 months. The patient’s spouse is the primary caregiver and is not aware of any adverse effects. The patient does not have a history of other psychiatric diagnoses such as schizophrenia or bipolar disorder.
Which one of the following would be the most appropriate intervention at this time?
A) Continue quetiapine at the current dosage
B) Reduce quetiapine to a lower maintenance dosage
C) Taper the quetiapine dosage with the goal of stopping it
D) Start diphenhydramine (Benadryl) while tapering quetiapine with the goal of stopping it
E) Start lorazepam (Ativan) while tapering quetiapine with the goal of stopping it
C) Taper the quetiapine dosage with the goal of stopping it
Behavioral and psychological symptoms of dementia include delusions, hallucinations, aggression, and agitation. Antipsychotics are frequently used for treatment of these symptoms and are continued indefinitely. For patients who have been taking antipsychotics for >3 months and whose symptoms have stabilized, or for patients who have not responded to an adequate trial of an antipsychotic, it is recommended that the drug be tapered slowly (SOR B).
Physicians should collaborate with the patient and caregivers when deciding whether to use an antipsychotic. This is recommended because antipsychotic medications have adverse effects, including an increased overall risk of death, cerebrovascular events, extrapyramidal symptoms, gait disturbances, falls, somnolence, edema, urinary tract infections, weight gain, and diabetes mellitus. The risk of these harms increases with prolonged use in the elderly.
One tapering method to consider is to reduce the daily dose to 75%, 50%, and 25% of the original dose every 2 weeks until stopping the medication. This reduction pace can be slowed for some patients.
- Diphenhydramine and lorazepam are on the Beers list of potentially inappropriate medications to use in older patients and would not be recommended.
2019.8)
A healthy 35-year-old female presents to your office to discuss an upcoming trip to Bangladesh. She currently feels well and has no health problems. She is a nurse and will be traveling with a church group to work in a clinic for 1 month. This area is known to have a high prevalence of tuberculosis (TB). She is worried about contracting TB while she is there and asks for recommendations regarding TB screening. She had a negative TB skin test about 1 year ago at work. A TB skin test today is negative.
Assuming she remains asymptomatic, which one of the following would you recommend?
A) Prophylactic treatment with isoniazid starting 1 month prior to departure and continuing throughout her trip
B) Prophylactic treatment with rifampin (Rifadin) starting 1 month prior to departure and continuing throughout her trip
C) A repeat TB skin test 2 months after she returns
D) A chest radiograph 2 months after she returns
E) An interferon-gamma release assay (IGRA) 6 months after she returns
C) A repeat TB skin test 2 months after she returns
Individuals who travel internationally to areas with a high prevalence of tuberculosis (TB) are at risk for contracting the disease if they have prolonged exposure to individuals with TB, such as working in a health care setting. The CDC recommends either a TB skin test or an interferon-gamma release assay prior to leaving the United States. If the test is negative, the individual should repeat the testing 8–10 weeks after returning.
- A chest radiograph in asymptomatic individuals or prophylactic treatment at any point is not recommended.
- Isoniazid and rifampin are options for treatment of latent TB.
2019.9)
A nulliparous 34-year-old female comes to your office for evaluation of fatigue, hair loss, and anterior neck pain. These symptoms have been gradually worsening for the past few months. Her past medical history is unremarkable. She has gained 5 kg (11 lb) since her last office visit 18 months ago. Examination of the thyroid gland reveals tenderness but no discrete nodules. Her TSH level is 7.5 U/mL (N 0.4–4.2), her T4 level is low, and her thyroid peroxidase antibodies are elevated.
Which one of the following would be the most appropriate next step?
A) Continue monitoring TSH every 6 months
B) Begin thyroid hormone replacement and repeat the TSH level in 6–8 weeks
C) Begin thyroid hormone replacement and repeat the TSH level along with a T3 level in 6–8 weeks
D) Order ultrasonography of the thyroid
E) Order fine-needle aspiration of the thyroid
B) Begin thyroid hormone replacement and repeat the TSH level in 6–8 weeks
This patient has thyroiditis with biochemical evidence for autoimmune (Hashimoto’s) thyroiditis. The most appropriate plan of care is to begin thyroid hormone replacement and monitor with a repeat TSH level 6–8 weeks later.
- It is not necessary to include a T3 level when assessing the levothyroxine dose.
- There is no need to routinely order thyroid ultrasonography when there are no palpable nodules on a thyroid examination.
- Fine-needle aspiration may be necessary to rule out infectious thyroiditis when a patient presents with severe thyroid pain and systemic symptoms.
2019.10)
A 35-year-old male presents with depression that started when his wife asked him for a divorce last month. A depression screen is positive and he has some passive suicidal ideation. He does not have any prior history of suicide attempts or a specific plan. He does not have any health issues, a family history of mental health issues, or a history of adverse childhood events.
You would be most concerned that the patient will die from suicide if he
A) has limited support from his family
B) has no religious affiliation
C) has a history of “cutting” as an adolescent
D) has easy access to firearms
E) was hospitalized for an appendectomy 2 months ago
D) has easy access to firearms
Easy access to a lethal means of suicide is a major risk factor for a successful suicide attempt. It is important to eliminate access to firearms, drugs, or toxins for a patient with any suicidal ideation. Other risk factors include, but are not limited to, a family history of suicide, previous suicide attempts, a history of mental disorders, a history of alcohol or substance abuse, and physical illness. Another risk factor in this patient is loss of a personal relationship. A history of borderline personality disorder (associated with cutting) is not a risk for successful suicide. Any support from family or friends is helpful, even if it is limited.
2019.11)
A 49-year-old African-American male sees you for a routine health maintenance examination. His past medical history is significant for sarcoidosis. He has noticed some fatigue and shortness of breath over the last several months, but he is asymptomatic today. His vital signs are normal except for an irregular pulse. An EKG performed in the office is shown below.
Which one of the following would be most appropriate at this point?
A) Observation only
B) Amiodarone (Cordarone)
C) Apixaban (Eliquis)
D) Metoprolol succinate (Toprol-XL)
E) A cardiology assessment for placement of a pacemaker
E) A cardiology assessment for placement of a pacemaker
This patient’s EKG shows type II second degree (Mobitz type II) atrioventricular (AV) block. Conduction disturbances are one of the most common manifestations of cardiac sarcoidosis. In addition to AV block, supraventricular and ventricular arrhythmias can be seen. Mobitz type II AV block is treated with pacemaker placement.
- Metoprolol could be used for treatment of nonsustained ventricular tachycardia,
- Apixaban for anticoagulation in patients with atrial fibrillation or atrial flutter, and
- Amiodarone for either supraventricular or ventricular tachycardias.
2019.12)
A 70-year-old male presents to your office for follow-up after he was hospitalized for acute coronary syndrome. He has not experienced any pain since discharge and is currently in a supervised cardiac rehabilitation exercise program. His medications include aspirin, lisinopril (Prinivil, Zestril), and metoprolol, but he was unable to tolerate atorvastatin (Lipitor), 40 mg daily, because he developed muscle aches.
Which one of the following would you recommend?
A) Evolocumab (Repatha)
B) Ezetimibe/simvastatin (Vytorin)
C) Fenofibrate (Tricor)
D) Niacin
E) Omega-3 fatty acid supplements
B) Ezetimibe/simvastatin (Vytorin)
High-intensity statin therapy is recommended for patients younger than 75 years of age with known coronary artery disease. For those who are intolerant of high-intensity statins, a trial of a moderate-intensity statin is appropriate. There is evidence to support ezetimibe plus a statin in patients with acute coronary syndrome or chronic kidney disease.
- Omega-3 fatty acids, fibrates, and niacin should not be prescribed for primary or secondary prevention of atherosclerotic cardiovascular disease because they do not affect patient-oriented outcomes.
- PCSK9 inhibitors such as evolocumab are injectable monoclonal antibodies that lower LDL-cholesterol levels significantly and have produced some promising results, but more studies are needed to determine when this would be cost effective.
2019.13)
A 50-year-old male presents with difficulty straightening his left ring finger. Examination of the affected hand reveals a nodule of the palmar aponeurosis and associated fibrous band that limits full extension of the fourth finger. He is unable to fully extend both the metacarpophalangeal (MCP) joint and the proximal interphalangeal (PIP) joint, with MCP and PIP contractures estimated at 40° and 20°, respectively.
Which one of the following would be the most appropriate management strategy?
A) Observation until the PIP contracture is >90°
B) Serial intralesional injection with a corticosteroid
C) Cryosurgery of the fibrous nodule
D) Referral for physical therapy
E) Referral for surgical release of the contracture
E) Referral for surgical release of the contracture
This patient has Dupuytren’s disease with a contracture of the affected finger. Surgical release is indicated when the MCP joint contracture reaches 30° or with any degree of contracture of the PIP joint.
- Intralesional injection may reduce the need for later surgery in a patient with grade 1 disease, but not if there is a contracture.
- There is no evidence to support the use of physical therapy or cryosurgery.
2019.14)
A 44-year-old female presents for a pretravel consultation and asks about medication options for traveler’s diarrhea. She will be on an organized tour traveling to a country with a very low risk for this problem. She plans to take all precautions to further reduce her risk but would also like you to recommend a medication she can take.
Which one of the following would be an appropriate recommendation?
A) A short course of azithromycin (Zithromax) if she develops diarrhea
B) Loperamide (Imodium) daily, starting 1 day prior to travel and continued until 1 day after returning home
C) Probiotics daily, starting 1 week prior to travel and continued until 1 week after returning home
D) Ciprofloxacin (Cipro) daily, starting 2 weeks prior to travel and continued until 4 weeks after returning home
E) Bismuth subsalicylate daily, starting 2 weeks prior to travel and continued until 4 weeks after returning home
A) A short course of azithromycin (Zithromax) if she develops diarrhea
Traveler’s diarrhea is the most common infection in international travelers. A short course of antibiotics can be taken after a traveler develops diarrhea and usually shortens the duration of symptoms (SOR A). Azithromycin is preferred to treat severe traveler’s diarrhea.
- Rifaximin or fluoroquinolones may be used to treat severe nondysenteric traveler’s diarrhea.
- Prophylactic antibiotics are not routinely recommended.
- For patients at high risk, bismuth subsalicylate reduces the risk but does not need to be initiated prior to travel.
- There is insufficient evidence for the use of probiotics to prevent traveler’s diarrhea.
- Loperamide can be used with or without antibiotics after symptoms develop but is not recommended for prophylaxis.
2019.15
A 69-year-old female presents to your office with a 5-day history of cough and low-grade fever. She has a past history of hypertension and obstructive sleep apnea. Her daughter brought her in this morning because of worsening symptoms. The patient’s temperature is 37.4°C (99.3°F), her blood pressure is 110/74 mm Hg, her pulse rate is 88 beats/min, her respiratory rate is 36/min, and her oxygen saturation is 95% on room air. She is alert and oriented to person, place, and time. A CBC and basic metabolic panel are normal except for an elevated WBC count of 12,500/mm3 (N 4300–10,800). A chest radiograph shows a right lower lobe infiltrate.
This patient has a higher risk of mortality and should be considered for inpatient treatment due to her
A) female sex
B) underlying hypertension
C) respiratory rate
D) elevated WBC count
E) abnormal chest radiograph
C) respiratory rate (REVIEW: 2017.17 )
CURB-65
Confusion (1pt)
Urea (BUN) >20 mg/dl
Resp rate >30 breath/min
BP <80/60 mmHg
Age >65
There are several decision support tools to assist in predicting 30-day mortality for patients with community-acquired pneumonia. Calculating the number of high-risk markers can aid in deciding whether to admit the patient to the hospital. The risk of mortality increases with a respiratory rate >30/min, hypotension, confusion or disorientation, a BUN level >20 mg/dL, age >65 years, male sex, or the presence of heart failure or COPD.
PPC 7-3
2019.16)
A 78-year-old male is brought to your office by his daughter. She is concerned that her father is no longer attending his weekly cribbage and bingo games, has stopped bathing regularly, and is eating much less.
Which one of the following would be most appropriate at this time?
A) Administering the CAGE screening questionnaire
B) Administering the PHQ-9 screening questionnaire
C) A trial of megestrol
D) A trial of nortriptyline (Pamelor)
E) MRI of the brain
B) Administering the PHQ-9 screening questionnaire
This elderly patient is exhibiting classic signs of depression. The PHQ-2 has a similar sensitivity to the PHQ-9, but the PHQ-9 has a higher specificity in diagnosing depression (91%–94% compared to 78%–92%) and can assist in diagnosing depression. In addition to the PHQ-2 and PHQ-9 there are specific screening tools for use in the elderly population, including the Geriatric Depression Scale and the Cornell Scale for Depression in Dementia. Somatic issues and dementia can make it more difficult to screen for and diagnose depression in this population.
- The CAGE questionnaire screens for substance abuse.
- Megestrol is used to stimulate the appetite, but in this patient the appetite symptoms are likely secondary to depression so treating the depression would be a more appropriate starting point.
- The tricyclic nortriptyline is used to treat depression but is not first-line therapy, especially in the elderly.
- In general, a more extensive medical history and a physical examination are indicated before ordering MRI of the brain.
2019.17)
The U.S. Preventive Services Task Force (USPSTF) recommends which one of the following for prevention of falls in community-dwelling adults >65 years of age who are at increased risk for falls?
A) Empirical vitamin D supplementation
B) Psychological evaluation and treatment programs
C) In-home environmental evaluation and modification
D) Regular participation in an exercise program
D) Regular participation in an exercise program
The U.S. Preventive Services Task Force (USPSTF) recommends exercise interventions to prevent falls in community-dwelling adults >65 years of age who are at increased risk for falls (B recommendation). This recommendation is based on several studies that demonstrated improved fall-related outcomes for individuals from this population who participated in exercise programs. Strength and resistance exercises were specifically identified as beneficial. The evidence exists to support group-based exercises is less convincing.
It is also recommended that clinicians selectively offer multifactorial interventions to prevent falls in this population, based on the possible small benefit and minimal risk (C recommendation). The USPSTF recommends against vitamin D supplementation to prevent falls in community-dwelling adults >65 years of age with the caveat that this applies only to those who are not known to have osteoporosis or vitamin D deficiency (D recommendation).
2019.18)
A 30-year-old gravida 1 para 0 develops erythematous patches with slightly elevated scaly borders during her first trimester. There was a 2-cm herald patch 2 weeks before multiple smaller patches appeared. The rash on the back has a “Christmas tree” pattern. She has not had any prenatal laboratory work.
This condition is associated with
A) no additional pregnancy risk
B) a small-for-gestational-age newborn
C) congenital cataracts
D) multiple birth defects
E) spontaneous abortion
E) spontaneous abortion
This patient has classic pityriasis rosea. This is generally a benign disease except in pregnancy. The epidemiology and clinical course suggest an infectious etiology. Pregnant women are more susceptible to pityriasis rosea because of decreased immunity.
Pityriasis rosea is associated with an increased rate of spontaneous abortion in the first 15 weeks of gestation.
- It is not associated with an increased risk for a small-for-gestational-age newborn, congenital cataracts, or multiple birth defects.
2019.19)
A 57-year-old male with diabetes mellitus and hypertension presents with a 1-month history of pain in his hands and elbows. His hands are shown below. On examination they are tender and he has soft swelling of the wrists, metacarpophalangeal (MCP) joints, and proximal interphalangeal (PIP) joints. Plain films show mild, diffuse bony erosions in the MCP and PIP joints.
Which one of the following is the most likely diagnosis?
A) Dermatomyositis
B) Osteoarthritis
C) Psoriatic arthritis
D) Rheumatoid arthritis
E) Systemic lupus erythematosus
D) Rheumatoid arthritis
This patient’s clinical findings and radiographs indicate a diagnosis of inflammatory arthritis, most likely rheumatoid arthritis:
- Symmetric
- Small-joint
- Inflammatory
- All are typical of rheumatoid arthritis and systemic lupus erythematosus (SLE), but bony erosions are not seen in SLE.
- Psoriatic arthritis can also affect small joints but is typically not symmetric.
- Dermatomyositis can present with a thick, bright red rash over the metacarpophalangeal (MCP) and interphalangeal joints (Gottron’s sign) but is typically associated with proximal muscle weakness rather than joint pain or erosions that can be seen on radiographs.
- Osteoarthritis does not typically cause the soft-tissue swelling seen in the image. It usually affects the distal and proximal interphalangeal joints while sparing the MCP joints, and it results in osteophytes and joint space narrowing that can be seen on radiographs.
2019.20)
A 77-year-old Spanish-speaking female with end-stage heart failure has elected hospice care to be provided at home for the duration of her life. A trained interpreter is available for assistance when you see the patient and is present in the room.
Which one of the following is considered a best practice when using interpreters?
A) Addressing the patient directly when speaking
B) Seating the interpreter closest to the clinician, slightly in front of the patient, to observe body language when translating
C) Asking the interpreter to serve as a witness for a consent form for hospice
D) Explaining to the interpreter the entire care plan, then having him or her repeat it back to the patient
E) Explaining in full detail all possible scenarios for symptom management and what to expect
A) Addressing the patient directly when speaking
When professional interpreters participate in patient care it is important to speak directly in the first person, using “I” statements rather than statements that start with “tell her” (SOR C). It is ideal to seat the interpreter next to or slightly behind the patient, so that the patient is the focus of the interaction. Sentence-by-sentence interpretation can prevent miscommunication errors, as opposed to expecting the interpreter to remember every detail of a complex care plan. It is not appropriate for the medical interpreter to also serve as a witness to consent. Focusing on three or fewer key points rather than over-communicating multiple complex issues increases the likelihood that the patient will comprehend the plan of care.
2019.21
Which one of the following treatments has been shown to improve the quality of life for a patient with tinnitus?
A) Antidepressant therapy
B) Ginkgo biloba
C) Niacin
D) Vitamin B12
E) Cognitive-behavioral therapy
E) Cognitive-behavioral therapy (REVIEW: 2018.144 )
Treatments to reduce awareness of tinnitus and tinnitus-related distress include cognitive-behavioral therapy, acoustic stimulation, and educational counseling.
No medications, supplements, or herbal remedies have been shown to substantially reduce the severity of tinnitus.
2019.22
A 28-year-old female who was recently diagnosed with polycystic ovary syndrome presents to discuss treatment of irregular menses. She has 2–3 menstrual periods every 6 months that happen at irregular times and can often produce heavy bleeding. She is not obese and has no significant acne or hirsutism. She does not desire pregnancy and her primary goal is to decrease the heavy menstrual bleeding.
Which one of the following would be the most effective initial recommendation?
A) Dietary modifications aimed at weight loss
B) Clomiphene
C) Metformin (Glucophage)
D) Spironolactone (Aldactone)
E) Placement of a levonorgestrel IUD (Mirena)
E) Placement of a levonorgestrel IUD (Mirena) (REVIEW: 2018.60 )
Polycystic ovary syndrome can significantly affect multiple organ systems, and menstrual irregularities from anovulatory cycles are very common. Treatment should be based on the patient’s goals and modified based on her desire for fertility.
- In a patient who is not interested in near-term fertility and whose goal is to control menstrual irregularities, a levonorgestrel IUD is most likely to reduce the frequency, duration, and volume of bleeding.
- Metformin is used to treat insulin resistance, dietary modifications are used to treat obesity,
- Spironolactone can be used to treat hirsutism or acne, and
- Clomiphene is used to induce ovulation and fertility.
2019.23
A 6-month-old male is brought to the urgent care center with a 3-day history of rhinorrhea, cough, and increased respiratory effort. His temperature is 37.5°C (99.5°F), his heart rate is 120 beats/min, his respiratory rate is 42/min, and his oxygen saturation is 96% on room air. On examination the child appears well hydrated with clear secretions from his nasal passages, there is diffuse wheezing heard bilaterally, and there is no nasal flaring or retractions. The mother states that the child has a decreased appetite but is drinking a normal amount of fluids.
Which one of the following would be the most appropriate management for this patient?
A) Supportive therapy only
B) Bronchodilators
C) A corticosteroid taper
D) Epinephrine
E) Nebulized hypertonic saline
A) Supportive therapy only (REVIEW: 2018.199 )
This patient’s symptoms and the examination suggest viral bronchiolitis. Supportive therapy, including adequate hydration, is recommended for treatment. Treatment with bronchodilators, epinephrine, hypertonic saline, or corticosteroids is not indicated (SOR A).
2019.24)
In asymptomatic patients with sarcoidosis, which one of the following organ systems should be examined yearly to detect extrapulmonary manifestations of the disease?
A) Cardiac
B) Neurologic
C) Ocular
D) Integumentary
C) Ocular
Sarcoidosis has numerous extrapulmonary manifestations. Because inflammation of the eye can result in permanent impairment and is often asymptomatic, patients require yearly eye examinations as well as additional monitoring with disease flares.
- Although skin involvement is common it is usually readily apparent and rarely has serious sequelae.
- Cardiac sarcoidosis can potentially lead to progressive heart failure and sudden death, but evaluation is needed only in patients who are symptomatic.
- Similarly, evaluation for neurologic involvement is needed only in patients who are symptomatic.
2019.25)
In addition to a thorough history and physical examination, the routine evaluation of patients presenting with syncope should include
A) a CBC, comprehensive metabolic panel, TSH level, and urinalysis
B) orthostatic blood pressure measurements and an EKG
C) cardiac stress testing
D) echocardiography and Doppler ultrasonography of the carotid arteries
E) CT or MRI of the brain
B) orthostatic blood pressure measurements and an EKG
Orthostatic blood pressure measurement and an EKG are indicated in the routine evaluation of patients with syncope. All other testing should be directed by findings obtained in the history and on the physical examination.
2019.26)
A 67-year-old male presents for a Medicare wellness visit. He underwent basic laboratory work prior to the office visit. He is feeling well and does not have any concerns or symptoms. His blood pressure is 127/76 mm Hg, his heart rate is 64 beats/min, and he is afebrile. A comprehensive metabolic panel is unremarkable. A CBC shows the following results:
WBCs…………………………………………………… 7500/mm3 (N4100–10,900)
RBCs…………………………………………………….. 4.05 million/mm3 (N4.70–6.10)
Hemoglobin ………………………………………… 12.9 g/dL (N14.0–18.0)
Hematocrit …………………………………………… 39% (N42–52)
Mean corpuscular volume…………………… 82 um3 (N80–95)
Platelets………………………………………………… 197,000/mm3 (N130,000–448,000)
Which one of the following would be the most appropriate next step in the workup of this patient?
A) A serum ferritin level
B) A serum transferrin receptor–ferritin index
C) Oral iron supplementation, and a repeat CBC in 4 weeks
D) Referral to a gastroenterologist
A) A serum ferritin level (REVIEW: 2019.190 )
Anemia is often diagnosed incidentally on laboratory testing and is often asymptomatic. It is associated with increased morbidity and mortality in older adults, and is often caused by nutritional deficiencies, chronic kidney disease, occult blood loss from gastrointestinal malignancies, or chronic inflammation. However, in many patients the cause remains unknown. A detailed history and physical examination are indicated. In patients with normocytic or microcytic anemia, a serum ferritin level should be ordered. A low serum ferritin level is associated with iron deficiency and should be further evaluated so the underlying cause can be addressed.
- A serum transferrin-receptor–ferritin index should be determined for patients with a serum ferritin level between 46 and 100 ng/mL to distinguish between iron deficiency anemia and other types of anemia.
- Referring this patient to a gastroenterologist would not be indicated at this time.
2019.27)
A 57-year-old male presents with left posterior heel pain that started several weeks ago. An examination reveals a nodular appearance at the site of insertion of the Achilles tendon to the calcaneus, and local tenderness of the distal tendon.
Which one of the following would be the safest and most appropriate initial management?
A) Local injection with a corticosteroid
B) Local injection with platelet-rich plasma
C) Physical therapy with eccentric calf-strengthening exercises
D) Immobilization of the ankle in a cast or boot for 4–6 weeks
E) Surgical debridement of the calcification about the distal tendon
C) Physical therapy with eccentric calf-strengthening exercises (REVIEW: 2018.111 )
This patient has typical symptoms and findings of Achilles tendinopathy. The best management involves eccentric calf-strengthening exercises.
- A local injection with corticosteroids or with platelet-rich plasma is ineffective and may increase the risk of a tendon rupture.
- Immobilization and surgical debridement may be considered if more conservative therapies have failed.
2019.28)
A 35-year-old gravida 2 para 2 reports diminished sexual arousal since initiating antidepressant therapy with sertraline (Zoloft). She has normal menstrual cycles and does not have any other symptoms.
Which one of the following would you recommend?
A) Black cohosh, 40 mg daily
B) Bupropion (Wellbutrin), 150 mg twice daily
C) Ethinyl estradiol, 0.5 mg daily
D) Ospemifene (Osphena), 60 mg daily
E) A testosterone patch, 300 g applied twice weekly
B) Bupropion (Wellbutrin), 150 mg twice daily
Bupropion can improve antidepressant-related sexual arousal dysfunction (SOR B).
- Black cohosh is considered a safe alternative for treating menopausal vasomotor symptoms, but not for treating sexual arousal dysfunction in women who are premenopausal.
- Ethinyl estradiol may be taken to improve sexual dysfunction related to menopausal symptoms.
- Vaginal estrogen therapy is recommended over oral estrogen when vaginal dryness is the primary symptom.
- Ospemifene is indicated for dyspareunia related to vulvar and vaginal atrophy due to menopause.
- Testosterone has proven to be effective for treating menopause-related low sexual desire but the evidence is limited due to the lack of long-term data. The Endocrine Society recommends consideration of a 3- to 6-month course of testosterone specifically for postmenopausal women with low sexual desire.
2019.29)
A 67-year-old male presents to your office for evaluation of chronic redness, flaking, and discomfort of his eyelids. Additionally, his eyes feel irritated, dry, and sandpapery at times. He has had difficulties with these symptoms on and off throughout his life but they have worsened lately. He has not had any vision changes and does not wear contact lenses.
On examination his eyelids appear red and mildly swollen with yellow crusting at the bases of the eyelashes. You note bilateral mild conjunctival injection. Visual acuity is intact, as are pupil reactions and extraocular movements.
Which one of the following treatments is appropriate first-line therapy for this condition?
A) Warm compresses and gentle cleansing with a mild shampoo
B) Sodium sulfacetamide eye drops
C) Topical betamethasone
D) Oral acyclovir (Zovirax)
E) Oral cephalexin (Keflex)
A) Warm compresses and gentle cleansing with a mild shampoo
This patient has blepharitis, a chronic inflammation of the eyelids. Seborrhea is a common cause in older adults. In younger patients including children, colonization with Staphylococcus may be a contributing factor. Meibomian gland dysfunction is often part of this condition, contributing to a reduced quality of tear films, which leads to dry eyes and irritation. Other diagnoses to consider in this patient include conjunctivitis, preseptal cellulitis, and Sjögren’s syndrome.
- Conjunctivitis typically involves the conjunctiva and an eye discharge but less involvement of the eyelids is present.
- Cellulitis is an acute rather than chronic condition and involves more pain and swelling.
- Sjögren’s syndrome causes dry eye but not inflammatory changes of the lid.
The initial treatment of blepharitis consists of lid hygiene using warm compresses to remove dried secretions and debris. Mild shampoo can help in this process and aid in keeping the bacterial colonization load down. In severe or recalcitrant cases a topical antibiotic ointment may be applied to the lids. Oral antibiotics can be considered for more severe cases.
2019.30)
A patient with a BMI of 32 kg/m2 has type 2 diabetes that is currently controlled by lifestyle interventions, including moderate-intensity physical activity and healthy low-calorie meals. The patient asks about nonnutritive sweeteners, containing few or no calories.
According to the American Diabetes Association, which one of the following would be the most appropriate advice?
A) Sucrose (table sugar) is preferred
B) Nonnutritive sweeteners are acceptable to use
C) Nonnutritive sweeteners worsen glucose control
D) Sucralose-based sweeteners, such as Splenda, should be avoided
E) Sweeteners with aspartame, such as Equal, should be avoided
B) Nonnutritive sweeteners are acceptable to use
Nonnutritive sweeteners contain few or no calories. According to the American Diabetes Association, nonnutritive sweeteners may be acceptable to use instead of nutritive sweeteners such as sucrose. They should be used in moderation if they are used.
The use of nonnutritive sweeteners can help to reduce overall intake of carbohydrates and calories. They do not significantly affect glycemic control. Research is inconsistent regarding the effects of nonnutritive sweeteners on weight loss, but most systematic reviews and meta-analyses demonstrate a benefit.
There is no recommendation to avoid sucralose or aspartame in patients with type 2 diabetes. Beverages sweetened with sugar are associated with an increased risk of type 2 diabetes.
2019.31
A 32-year-old female presents with a 4-month history of nasal drainage, congestion, and loss of her sense of smell. She reports having a cold about 4 months ago that never resolved. On examination the nasal turbinates are swollen and you note mucopurulent drainage on the right.
Which one of the following is the most likely cause of her symptoms?
A) Chronic rhinosinusitis
B) Granulomatosis with polyangiitis (Wegener’s granulomatosis)
C) Nasal polyposis
D) Sarcoidosis
E) Seasonal allergic rhinitis
A) Chronic rhinosinusitis (REVIEW: 2018.215 )
The American Academy of Otolaryngology defines chronic rhinosinusitis as the presence of two of four cardinal symptoms, which include
- nasal drainage,
- nasal obstruction,
- facial pain or pressure, and
- hyposmia or anosmia, along with
- objective signs on examination or radiographic studies.
This patient has three cardinal symptoms of chronic rhinosinusitis and objective evidence on the physical examination.
- No nasal polyps were seen on the examination.
- Granulomatosis with polyangiitis and sarcoidosis can both present similarly but are uncommon causes of chronic rhinosinusitis.
- Allergic rhinitis can be associated with chronic rhinosinusitis but would also present with allergic symptoms.
2019.32)
A 52-year-old female with metastatic breast cancer is hospitalized for treatment of complications from her cancer treatment. She has developed a new onset of back pain that has been progressively worsening over the past few hours. The pain is worse when she is lying down and is not responsive to pain medication.
Which one of the following would be the most appropriate next step to address this patient’s back pain?
A) Increase the dosage of her immediate-release morphine
B) Increase the dosage of her sustained-release morphine
C) Order cyclobenzaprine
D) Order an urgent MRI
E) Order a physical therapy consultation for mobility
D) Order an urgent MRI
Malignant epidural spinal cord compression is an oncologic emergency that requires urgent MRI to confirm the diagnosis. It is caused by a tumor compressing the dural sac and should be suspected with new-onset progressive back pain that is worse when the patient is lying down. It is most commonly associated with breast cancer and develops in approximately 5% of all patients with cancer. Once the diagnosis is confirmed, an urgent management approach is needed.
- Corticosteroids and neurosurgical intervention can preserve motor and sensory function.
- Attempting to alleviate the pain would not address this emergency.
2019.33)
A 78-year-old female with Alzheimer’s disease is accompanied to an office visit by her daughter. The daughter has asked to complete an advance directive giving her medical power of attorney.
Which one of the following would indicate that the patient lacks capacity to make decisions with regard to completing her medical directive?
A) A dementing illness
B) Inconsistent answers to questions
C) Lack of orientation to time
D) Asking that her son make medical decisions for her instead of her daughter
E) A score of 24/30 on the Mini-Mental State Examination
B) Inconsistent answers to questions
In order for patients to show they have the capacity to make a decision they must demonstrate an understanding of the situation, including the risks, benefits, and consequences of the decision or refusal of care. If a patient gives inconsistent answers to questions after multiple explanations, this indicates that there is a lack of understanding and would meet one of the criteria to determine that the patient lacks the capacity to make that decision.
- The presence of dementia can be associated with an increased incidence of having a lack of capacity; however, a diagnosis of dementia by itself does not indicate that the patient lacks the capacity to make a decision.
- While disorientation to time or a lower score on the Mini-Mental State Examination is associated with an increased risk of lacking capacity, these findings alone would not be enough to determine that the patient lacks capacity.
- The patient asking that her son be her medical decision maker instead of her daughter would not be an indication that she lacks capacity.
2019.34
A 72-year-old male with a past history of hypertension, COPD, and pulmonary embolism presents with nonspecific symptoms including fatigue and syncope. You suspect he has pulmonary hypertension.
Which one of the following would be the most appropriate initial test?
A) Pulmonary function tests
B) Chest CT with contrast
C) Echocardiography
D) A coronary calcium scan
E) Right heart catheterization
C) Echocardiography
According to national guidelines echocardiography is the preferred initial noninvasive testing modality when pulmonary hypertension is suspected (SOR C).
- Pulmonary function tests provide helpful information in regard to pulmonary capacity but are not necessarily diagnostic of pulmonary hypertension.
- CT of the chest with contrast will not provide pulmonary pressures but may assist in the detection of pulmonary emboli.
- A coronary calcium scan may be indicated to evaluate for coronary artery disease but it is not a diagnostic test for pulmonary hypertension.
- Although right heart catheterization would provide pulmonary pressure values it is considered more invasive than echocardiography and is not always necessary for making the diagnosis.
2019.35
A 19-year-old female member of a college cross-country team presents with a 1-week history of right knee pain. She does not have any acute injury to the knee. An examination reveals no deformity and she has a normal gait. She has tenderness and subtle swelling localized 1 cm distal to the right medial joint line, and examinations of the knee and hip are otherwise normal.
Which one of the following is the most likely diagnosis?
A) Fibular head stress fracture
B) Iliotibial band syndrome
C) Medial meniscal tear
D) Pes anserine bursitis
E) Tibial apophysitis (Osgood-Schlatter disease)
D) Pes anserine bursitis
This patient has medial knee pain related to repetitive use, most likely caused by pes anserine bursitis. Iliotibial band syndrome is often related to overuse but causes pain in the lateral knee.
- The fibular head is also lateral to the knee joint.
- Osgood-Schlatter disease is also often related to overuse but causes pain at the insertion of the patellar ligament on the midline proximal tibia.
- A medial meniscal tear would localize to the medial joint line rather than distal to the joint line and would more likely be associated with positive findings from other examinations, such as a McMurray test.
2019.36
A 52-year-old female sees you because of a vaginal discharge. An examination reveals a malodorous, greenish-yellow, frothy discharge, and inflammation of the cervix and vagina.
Which one of the following is the most likely diagnosis?
A) Atrophic vaginitis
B) Irritant/allergic vaginitis
C) Bacterial vaginosis
D) Trichomoniasis
E) Vulvovaginal candidiasis
D) Trichomoniasis
Trichomoniasis classically presents as a greenish-yellow, frothy discharge with a foul odor. Erythema and inflammation of the vagina and cervix are often present and can include punctate hemorrhages (strawberry cervix).
- Atrophic vaginitis may cause a thin, clear discharge and is usually associated with a thin, friable vaginal mucosa.
- Irritant/allergic vaginitis causes burning and soreness with vulvar erythema but usually does not cause any significant discharge.
- Bacterial vaginosis more commonly presents as a thin, homogenous discharge with a fishy odor and no cervical or vaginal inflammation.
- Vulvovaginal candidiasis presents with white, thick, cheesy, or curdy discharge.
2019.37
The father of a healthy 14-year-old male calls you about a recent mumps outbreak in your community. The child never received the MMR vaccine because the parents declined the immunization despite extensive counseling about the topic.
You advise the father that
A) mumps typically starts with a cough, coryza, and conjunctivitis
B) mumps causes a pruritic rash with fluid-filled blisters
C) mumps can cause orchitis, possibly resulting in decreased fertility
D) Koplik spots or whitish papules in the mouth are pathognomonic for mumps
E) the MMR vaccine is not recommended for patients in this age range
C) mumps can cause orchitis, possibly resulting in decreased fertility
Prodromal symptoms of mumps include
- myalgia,
- fatigue,
- loss of appetite,
- fever, and
- headache.
- Parotitis is the most common manifestation.
- Infertility, meningitis, and encephalitis are serious complications of orchitis.
- Measles is characterized by
- cough,
- coryza,
- conjunctivitis, and
- Koplik spots.
- Varicella is characterized by a pruritic rash with fluid-filled blisters.
- MMR vaccine is indicated for this child.
2019.38
A 42-year-old female with diabetes mellitus comes to your office because of recurrent yeast infections. She is taking numerous agents in an attempt to lower her glucose level.
Which one of the following classes of antidiabetic agents is associated with an increased risk for candidiasis?
A) Biguanides such as metformin (Glucophage)
B) DPP-4 inhibitors such as sitagliptin (Januvia)
C) SGLT2 inhibitors such as empagliflozin (Jardiance)
D) GLP-1 receptor agonists such as liraglutide (Victoza)
E) Sulfonylureas such as glipizide (Glucotrol)
C) SGLT2 inhibitors such as empagliflozin (Jardiance) (REVIEW: 2018.136 )
SGLT2 inhibitors are known to cause an increased risk of yeast vaginitis because their mechanism of action involves blocking renal uptake of glucose, which results in an increase in glucosuria (SOR A).
- Common side effects of metformin include gastrointestinal upset.
- DPP-4 inhibitors have very few side effects.
- GLP-1 receptor agonists typically cause nausea and early satiety and weight loss.
- Sulfonylureas are associated with weight gain and hypoglycemia.
PCC 4-5
2019.39
A gravida 2 para 0 at 34 weeks gestation presents to your office because of diffuse itching. She does not have any known allergies other than seasonal allergies, and she does not have any new contacts. An examination is normal other than some scattered excoriations, and there is no other distinct rash. She has tried moisturizers but her symptoms have not improved.
Which one of the following would be most appropriate at this point?
A) Monitoring for the development of a rash
B) Liver function tests and serum bile acid levels
C) Topical corticosteroids
D) Oral antihistamines
E) Varicella-zoster immune globulin
B) Liver function tests and serum bile acid levels
Whenever a pregnant woman presents with pruritus without a primary rash, it is important to evaluate her for intrahepatic cholestasis of pregnancy. This diagnosis is associated with increased fetal mortality and warrants increased antenatal surveillance as well as possible induction by 35–37 weeks gestation. It is most appropriate to check for elevation of liver function tests and serum bile acids.
- Emollients, topical corticosteroids, and oral antihistamines can all be helpful for pruritus and certain rashes, but in this patient it is most important to promptly look for the cause of the pruritus.
- Varicella-zoster immune globulin would be indicated if she had no immunity to varicella and had been exposed to varicella or if she had a rash that was suspected to be chickenpox.
2019.40
When titrating the dosage of opioids, the CDC recommends that you should also consider prescribing naloxone when the opioid dosage reaches what morphine milligram equivalent (MME) per day threshold?
A) 30
B) 50
C) 80
D) 90
E) 100
B) 50
To mitigate the risk of opioid harm, it is essential to understand morphine milligram equivalents (MME). The evidence shows that the risk of an opioid overdose increases at the threshold of 50 MME/day. It is therefore recommended by the CDC that a prescription for naloxone be ordered when an opioid dosage reaches 50 MME/day, which is a high dosage.
- In general one should avoid prescribing >90 MME/day because of the substantially higher risk of an overdose at this dosage level.
2019.41
A 62-year-old female with stage 3 chronic kidney disease and an estimated glomerular filtration rate of 37 mL/min/1.73 m2 is found to have a mildly low ionized calcium level. Which one of the following would you expect to see if her hypocalcemia is secondary to her chronic kidney disease?
A) Elevated parathyroid hormone (PTH) and elevated phosphorus
B) Elevated PTH and low phosphorus
C) Low PTH and elevated phosphorus
D) Low PTH and low phosphorus
A) Elevated parathyroid hormone (PTH) and elevated phosphorus
Chronic kidney disease–mineral and bone disorder (CKD-MBD) is found in many patients with CKD and is associated with an increased risk of bone fractures and cardiovascular events due to vascular calcification. In patients with CKD, phosphate is not appropriately excreted and the subsequent hyperphosphatemia leads to secondary hyperparathyroidism and binding of calcium. Decreased production of calcitriol in patients with CKD also leads to hypocalcemic hyperparathyroidism. Patients with CKD stages 3a–5 should have phosphorus, calcium, parathyroid hormone, and 25-hydroxyvitamin D levels checked regularly, and consultation with a nephrologist or endocrinologist should be obtained if CKD-MBD is suspected.
2019.42
A 1-day-old newborn is brought to your office for a routine examination. His parents report that he is well. The prenatal course and delivery were unremarkable. An examination is normal except for a 1-cm wide dimple on the sacrum, 1 cm superior to the anus. The dimple has a tuft of dark hair.
At this point you would recommend
A) a follow-up examination in 1 month
B) ultrasonography
C) MRI
D) a fistulogram/sinogram
E) a dermatology consultation
B) ultrasonography
Recognizing clinically significant abnormalities on the newborn examination is important. Newborns with small sacral dimples located far from the anal verge, without other skin findings such as hair, do not need imaging to rule out spinal dysraphism (tethered cord). While the exact parameters of what is considered large (>0.5 cm diameter) and close (within 2.5 cm of the anal verge) can easily be found in reference materials, the dimple described here is clearly concerning and needs imaging. Ultrasonography can accurately and safely detect spinal dysraphism in these cases.
2019.43
A 63-year-old female sees you for evaluation of recurrent right foot swelling and redness. She has a history of obesity and type 2 diabetes with retinopathy, nephropathy, and peripheral neuropathy. She presented with similar symptoms 2 weeks ago and was diagnosed with cellulitis and treated with a 10-day course of amoxicillin/clavulanate (Augmentin). Her symptoms seemed to initially improve with this therapy along with elevation of the foot but then worsened. She does not have any pain in the foot, fever, or chills. She does not recall any trauma or other inciting event.
The patient’s vital signs include a temperature of 37.1°C (98.8°F), a pulse rate of 72 beats/min, and a blood pressure of 124/82 mm Hg. Her right foot appears swollen, red, and warm to the touch, and is not tender to palpation. There are no open sores or calluses. Her dorsalis pedis pulse is 2+. Monofilament testing confirms a diagnosis of peripheral neuropathy. A WBC count is normal. Radiographs reveal soft-tissue edema with no other abnormalities.
The most appropriate treatment at this point would be
A) immobilization
B) antibiotics
C) bisphosphonates
D) corticosteroids
E) surgical repair
A) immobilization
This patient has acute Charcot neuroarthropathy, an inflammatory condition that occurs in obese patients with peripheral neuropathy and ultimately leads to foot deformities (the classic rocker-bottom foot) and resultant ulcerations and infections. Its clinical appearance can easily be initially mistaken for cellulitis. However, the absence of tenderness and other signs of infection such as fever, an elevated WBC count, and inflammatory markers is not consistent with cellulitis. Radiography is an appropriate initial imaging modality but the results are often interpreted as normal early in the disease process. MRI is the modality of choice for a definitive diagnosis and may demonstrate periarticular bone marrow edema, adjacent soft-tissue edema, joint effusion, and microtrabecular or stress fractures.
The treatment of acute Charcot neuroarthropathy is immobilization with total contact casting, which increases the total surface area of contact to the entire lower extremity, distributing pressure away from the foot. Immobilization is typically required for at least 3–4 months but in some cases may be needed for up to 12 months. Bisphosphonates were found to be ineffective as adjunctive therapy in acute Charcot neuroarthropathy. Corticosteroids and antibiotics have no role in the treatment of Charcot foot but would be appropriate therapy for cellulitis or gout, which are important alternative diagnoses to consider. The role of surgery is more controversial but may be indicated in the acute phase of Charcot neuroarthropathy in patients with severe dislocation or instability.
2019.44
A 38-year-old female with a 6-month history of mild shortness of breath associated with some intermittent wheezing during upper respiratory infections presents for follow-up. You previously prescribed albuterol (Proventil, Ventolin) via metered-dose inhaler, which she says helps her symptoms. You suspect asthma. Pulmonary function testing reveals a normal FEV1/FVC ratio for her age.
Which one of the following would be the most appropriate next step?
A) Consider an alternative diagnosis
B) Assess her bronchodilator response
C) Perform a methacholine challenge
D) Prescribe an inhaled corticosteroid
E) Proceed with treatment for COPD
C) Perform a methacholine challenge
Spirometry is central to confirming the diagnosis of asthma, which is characterized by a reversible obstructive pattern of pulmonary function. In this case the patient’s FEV1/FVC ratio is normal, which neither confirms nor rules out asthma.
A methacholine challenge is recommended in this scenario to assess for the airway hyperresponsiveness that is the hallmark of asthma. Methacholine is a cholinergic agonist. Bronchoconstriction (defined as a reduction in FEV1 >20%) observed at low levels of methacholine administration (<4 mg/mL) is consistent with asthma.
If the FEV1/FVC ratio is reduced on initial spirometry, a bronchodilator response should be tested.
A fixed or partially reversible obstructive pattern suggests an alternative diagnosis such as COPD, and full reversal after bronchodilator use is consistent with asthma. Inhaled corticosteroids are not appropriate for intermittent asthma.
PPC 13-3
2019.45
An 80-year-old former smoker sees you for a 6-month follow-up for hypertension. He is taking carvedilol (Coreg), amlodipine (Norvasc), and low-dose aspirin. His home blood pressure readings have been 130–150/80–90 mmHg. Over the last 4 months he has developed pain in his thighs when walking to his mailbox a block away. The pain resolves after he sits for a few minutes.
On examination he has a blood pressure of 135/85 mm Hg, a heart rate of 72 beats/min, a BMI of 26 kg/m2, and an oxygen saturation of 95% on room air. Examinations of the heart and lungs are normal. There is dependent rubor of both legs but posterior tibial pulses are palpable. No ulcerations are noted. You obtain ankle-brachial indices of 0.85 on the left and 0.80 on the right. You prescribe a daily walking program.
Which one of the following additional measures would be most appropriate for this patient?
A) Add atorvastatin (Lipitor)
B) Add clopidogrel (Plavix)
C) Add lisinopril (Prinivil, Zestril) to achieve a goal blood pressure <120/80 mm Hg
D) Discontinue aspirin and start warfarin (Coumadin)
E) Refer to a vascular surgeon
A) Add atorvastatin (Lipitor)
Management of asymptomatic peripheral artery disease (PAD) should initially be conservative and should include a walking program (SOR A), smoking cessation, and a healthy diet.
- Statins should be started for all patients with PAD regardless of their LDL-cholesterol levels (SOR A).
- High-intensity statins should be used if tolerated.
- A single antiplatelet agent is recommended for patients with PAD. Both aspirin and clopidogrel are effective in the reduction of stroke, but the combination of the two is recommended only after revascularization surgery.
- Blood pressure control is indicated in patients with PAD but no antihypertensive class is clearly superior to another, although there is some evidence that ACE inhibitors may have additional benefits in terms of walking and pain. In an 80-year-old patient, lowering blood pressure below 120/80 mm Hg can be associated with significant side effects, including a greater risk of falls.
- Anticoagulants have not been shown to reduce the risk of major cardiovascular events in patients with PAD and they increase the risk of life-threatening bleeding.
- Referral to a vascular surgeon or for angiography is indicated if conservative therapy fails or symptoms worsen acutely, pain occurs at rest, or the patient develops ulcerations or loss of tissue.
2019.46
An 84-year-old male nursing home resident with dementia is noted to have a weight loss of about 5% in the past 6 months. Which one of the following would be most appropriate?
A) Avoiding dietary restrictions
B) An appetite stimulant
C) Vitamin B12, vitamin D, and selenium supplements
D) An omega-3 fatty acid supplement
E) Tube feeding
A) Avoiding dietary restrictions
Effective interventions for weight loss in nursing home patients include providing meals in a pleasant, home-like environment. Avoiding dietary restrictions has low quality evidence of effectiveness. There is high quality evidence that initiating tube feedings in patients with severe dementia is not only ineffective but may lead to problems such as decubitus ulcers and aspiration. There is low to very low evidence of the effectiveness for prescribing appetite stimulants, selenium, vitamin B, or vitamin D supplements unless there is a documented deficiency. Neither quality of life nor survival is improved.
2019.47
According to the most recent American College of Cardiology/American Heart Association guidelines, hypertension is defined as a blood pressure reading greater than
A) 120/80 mm Hg
B) 130/80 mm Hg
C) 135/85 mm Hg
D) 140/90 mm Hg
E) 150/90 mm Hg
B) 130/80 mm Hg
The latest American College of Cardiology/American Heart Association guidelines promote a radical change in the management of hypertension, which they now define as a blood pressure >130/80 mmHg.
- 120–129 / <80 mmHg: Elevated blood pressure
- 130–139 / 80–89 mmHg: stage 1 hypertension
- >140 / >90 mmHg: stage 2 hypertension.
2019.48
A 26-year-old G2P1001 at 30 weeks gestation was recently diagnosed with gestational diabetes and is ready to start testing her blood glucose at home. Which one of the following is the recommended goal for fasting blood glucose in this patient?
A) <75 mg/dL
B) <95 mg/dL
C) <120 mg/dL
D) <150 mg/dL
E) <180 mg/dL
B) <95 mg/dL
The goal fasting blood glucose level in patients with gestational diabetes is <95 mg/dL.
- A fasting glucose level <80 mg/dL is associated with increased maternal and fetal complications.
- The goal 2-hour postprandial glucose level is <120 mg/dL and the
- Goal 1-hour postprandial glucose level is <140 mg/dL.
2019.49
A 67-year-old male diagnosed with polymyalgia rheumatica is started on long-term prednisone therapy. Which one of the following is the recommended first-line agent to prevent steroid-induced osteoporosis?
A) Alendronate (Fosamax)
B) High-dose vitamin D
C) Raloxifene (Evista)
D) Teriparatide (Forteo)
A) Alendronate (Fosamax) (REVIEW: 2018.207 )
Patients are at risk of developing glucocorticoid-induced osteoporosis if they are on long-term glucocorticoid therapy, defined as >2.5 mg of prednisone for a duration of 3 months or longer.
The American College of Rheumatology recommends pharmacologic treatment for these patients, as well as for patients receiving glucocorticoids who have a
- bone mineral density T-score <–2.5 at either the spine or the femoral neck and are either
- male and >50 years of age or
- female and postmenopausal.
Therapy is also recommended in patients >40 years of age who do not meet these criteria but have a 10-year risk of major osteoporotic fracture of at least 20% or a risk of hip fracture of at least 3% according to the FRAX tool.
Oral bisphosphonates are recommended as first-line agents for preventing glucocorticoid-induced osteoporotic fractures, although intravenous bisphosphonates can be used if patients are unable to use the oral forms.
Supplementation of calcium (800–1000 mg) and vitamin D (400–800 IU) is also recommended.
Raloxifene and teriparatide are options when bisphosphonate therapy fails or is contraindicated (SOR A).
2019.50
At a routine well child check, the mother of an 18-month-old female expresses concern about the child’s development. Which one of the following should prompt consideration of a developmental delay?
A) A vocabulary of less than six words
B) Failure to point to pictures or body parts when named
C) Inability to follow one-step directions
D) Inability to run well
E) Inability to copy a vertical line
C) Inability to follow one-step directions
At 18 months of age a child should follow one-step directions.
- Approximately 90% of 18-month-olds say at least three words, and 50%–90% say six words.
- The ability to point to body parts or pictures after they are named is expected at 2 years of age.
- Not walking at 18 months would be a red flag for delay, but running well may not yet be accomplished.
- At 18 months a child would be expected to scribble spontaneously but not to copy a vertical line.
2019.51
Which one of the following is the strongest indication for formal allergy testing?
A) Erythema and tenderness surrounding an insect sting for 24 hours
B) A fever for 3 days followed by a diffuse urticarial rash in a child
C) A diffuse whole-body rash following ingestion of trimethoprim/sulfamethoxazole (Bactrim) in a patient with no documented drug allergies
D) Recurrent or persistent upper respiratory symptoms
E) Persistent epigastric pain following ingestion of tomato products
D) Recurrent or persistent upper respiratory symptoms
Despite 10%–30% of the population being affected by allergic disease, allergy testing does have limitations and is most useful in certain clinical situations. Allergy testing can be helpful in patients with persistent sinus infections, allergic rhinitis, and poorly controlled asthma.
- Allergy testing for insect stings is indicated only following systemic/anaphylactic or large local reactions, not with limited localized reactions.
- Three days of fever followed by a diffuse urticarial rash likely represents a rash associated with a limited viral illness.
- Allergy testing for penicillin has a negative predictive value of 95%–98%. Testing for allergy to other antibiotics has a much lower sensitivity and specificity but does have limited use to help guide medication choices in patients with multiple allergies and when limited antibiotic options are available.
- Persistent epigastric pain following the ingestion of tomato products is more indicative of acid reflux symptoms rather than a tomato allergy.
2019.52
A 22-year-old male presents to your office the morning after falling onto his outstretched right hand as he tripped while leaving a bar. He has a deep, dull ache in the right wrist on the radial side. The pain is worsened by gripping and squeezing. On examination there is some wrist fullness and the wrist is tender to palpation over the anatomic snuffbox. Radiographs of the wrist are negative.
Which one of the following would be most appropriate at this time?
A) Rest, ice, compression, elevation, and NSAIDs with no specific follow-up
B) Rest, ice, compression, elevation, and NSAIDs with a follow-up examination in 2 weeks
C) Placement of a thumb spica splint, with a follow-up examination in 2 weeks
D) CT of the wrist to detect an occult fracture
E) Ultrasonography of the wrist to detect a ligament injury
C) Placement of a thumb spica splint, with a follow-up examination in 2 weeks
The history, symptoms, and physical examination findings in this case suggest a scaphoid fracture. The scaphoid bone is the most commonly fractured carpal bone and a fall on an outstretched hand can produce enough force to cause this fracture. This fracture is most common in males 15–30 years of age.
The finding of anatomic snuffbox tenderness is highly sensitive but not specific for a scaphoid fracture. Initial radiographs often do not demonstrate a fracture. When there is a high clinical suspicion for a scaphoid fracture but radiographs are negative, it is reasonable to immobilize in a thumb spica splint and reevaluate in 2 weeks.
Treatment for a sprain with or without follow-up would not be ideal in a situation where a scaphoid fracture is suspected. MRI or bone scintigraphy can be considered if the patient desires or needs an immediate diagnosis, but CT and ultrasonography are not appropriate imaging modalities for this fracture.
2019.53
A 54-year-old male develops chest pain while running. He is rushed to the emergency department of a hospital equipped for percutaneous coronary intervention. An EKG shows 3 mm of ST elevation in the anterior leads. He is diaphoretic and cool with ongoing chest pain. His blood pressure is 80/50 mm Hg, his pulse rate is 116 beats/min, and his oxygen saturation is 98% on room air.
You would immediately administer
A) a ß-blocker
B) dual antiplatelet therapy and an anticoagulant
C) intravenous fibrinolytic therapy
D) an intravenous vasopressor
B) dual antiplatelet therapy and an anticoagulant
This patient is likely experiencing an acute anterior wall myocardial infarction with possible incipient cardiogenic shock. Along with initiating the hospital’s protocol for myocardial infarction, immediate treatment should include
- Dual antiplatelet therapy with a 325-mg dose of nonenteric aspirin, a
- P2Y12 inhibitor (clopidogrel, prasugrel, or ticagrelor), and an
- Anticoagulant (unfractionated heparin or bivalirudin).
Given the possibility of cardiogenic shock, ß-blockers should not be used. Unless more than a 2-hour delay in percutaneous coronary intervention is expected, fibrinolytics should not be administered. An intravenous vasopressor is not indicated.
2019.54
A 38-year-old female presents for follow-up of a second hospitalization in the past 3 months for acute hepatitis, thrombocytopenia, and alcohol withdrawal symptoms treated with benzodiazepines. She says that prior to her hospitalization a week ago she had been drinking a half pint of vodka daily. She reports that her drinking has gradually increased over the past 10 years but increased significantly 6 months ago after she lost her job at a bar and grill. She knows her alcohol consumption is causing damage to her liver and tells you that her aunt died of alcoholic cirrhosis this year. Despite this knowledge she does not want to stop drinking at this time. She has looked into several alcohol cessation programs in the area but does not think that they are a good fit for her.
She currently lives with her boyfriend who also uses alcohol and cocaine regularly. She is not currently speaking to her mother because they “don’t see eye to eye.” She tells you that she has not consumed alcohol since her discharge from the hospital 2 days ago. She reports that her abdominal pain, nausea, and vomiting have resolved and she is feeling well.
The most likely diagnosis is
A) alcohol intoxication
B) alcohol withdrawal
C) alcohol use disorder in early remission
D) severe alcohol use disorder
D) severe alcohol use disorder
This patient presents with 6 out of 11 symptoms of alcohol use disorder within a 12-month period, including
- A strong desire or urge to use alcohol,
- Recurrent alcohol use that has contributed to the inability to fulfill work obligations,
- Continued alcohol use despite interpersonal problems with her family,
- Continued alcohol use despite knowledge that it is causing physical damage to her liver,
- Development of a tolerance to the effects of alcohol over time, and
- Withdrawal symptoms that require treatment with benzodiazepines.
Mild alcohol use disorder is defined by the presence of 2–3 of the 11 symptoms documented in the DSM-5, whereas 3–5 symptoms indicate moderate alcohol use disorder and 6 or more symptoms indicate severe alcohol use disorder. This patient has severe alcohol use disorder that is currently active.
- Early remission is defined as the absence of symptoms for at least 3 months but less than 12 months.
- She is not currently intoxicated, and she does not currently have withdrawal symptoms related to her alcohol use over a week ago.
2019.55
A 67-year-old male presents to your office because of fatigue and a syncopal episode. His vital signs in the office are normal. An examination reveals a harsh systolic murmur best heard over the second right intercostal space radiating to the neck. Echocardiography confirms your suspected diagnosis.
Which one of the following is the only treatment that improves mortality with this condition?
A) ß-Blockers
B) Antimicrobial prophylaxis for bacterial endocarditis
C) Aortic valve replacement
D) Mitral valve repair
E) Ventricular septal defect closure
C) Aortic valve replacement (REVIEW: 2018.180 )
This patient has symptomatic severe aortic stenosis. The only treatment that improves this condition is aortic valve replacement (SOR B). Transcutaneous aortic valve replacement may be an alternative for patients who are not candidates for surgery.
ß-Blockers must be used with caution due to the risk of depressing left ventricular systolic function. They have not been shown to improve mortality.
Antimicrobial prophylaxis is not indicated unless a patient has undergone valve replacement or has a history of endocarditis (SOR C).
Atrial fibrillation is common in patients with aortic stenosis and rate control is important.
Symptomatic mitral valve regurgitation (MR) may require mitral valve intervention. However, these murmurs are
- holosystolic,
- high pitched, and
- best heard at the cardiac apex.
A ventricular septal defect (VSD) can cause a loud holosystolic murmur with an associated thrill heard best at the third/fourth interspace along the sternal border.
2019.56
A 28-year-old white female comes to your office at 37 weeks gestation with a 24-hour history of painful vesicles on the vulva. She does not have a past history of similar lesions. You make a presumptive diagnosis of genital herpes.
Of the following, the most sensitive and specific test is
A) exfoliative cytology (Tzanck test)
B) a polymerase chain reaction (PCR) test
C) an enzyme-linked immunosorbent assay (ELISA)
D) HSV serology (IgG/IgM)
B) a polymerase chain reaction (PCR) test
When genital herpes occurs during pregnancy, the best method of diagnosis is either a tissue culture or a polymerase chain reaction (PCR) test, which is more sensitive. Enzyme-linked immunosorbent assays are sensitive, but not as sensitive or specific as PCR.
2019.57
A 27-year-old white female with a history of mania sees you because of polyuria and increased thirst over the past month. She has taken lithium, 1800 mg daily, for 3 years and her mania is well controlled. She has not lost weight and there is no family history of her current problem. There are no orthostatic blood pressure changes.
Laboratory Findings
Serum sodium…………….. 145mEq/L(N135–145)
Serum potassium…………. 4.5mEq/L(N3.5–5.0)
Serum glucose…………….. 92mg/dL
Serum creatinine…………. 0.9mg/dL(N0.6–1.5)
Serum lithium………………. 1.38mEq/L (therapeutic range 0.5–1.5)
Urine volume……………….. 6.85 L/24 hr
Urine osmolality………….. 161mOsm/kgH2O
There is no significant change in urine osmolality in response to the administration of vasopressin. Which one of the following is the most likely cause of this patient’s problem?
A) Drug-induced nephrogenic diabetes insipidus
B) Borderline diabetes mellitus
C) Panhypopituitarism
D) Psychogenic water drinking
A) Drug-induced nephrogenic diabetes insipidus (REVIEW: 2018.139 )
Polyuria occurs in 20%–70% of patients on long-term lithium therapy, even when plasma lithium levels are in the therapeutic range. This is a result of impaired renal concentrating ability that is resistant to vasopressin (nephrogenic diabetes insipidus).
- Inappropriate antidiuretic hormone secretion causes hyponatremia and fluid retention.
- The diuresis associated with diabetes mellitus is a result of the osmotic effect of increased serum glucose, which is not present in this case.
- Patients with hypothalamic or pituitary injuries may develop central diabetes insipidus, which responds to exogenous vasopressin.
- Psychogenic water drinking occurs in psychiatric patients, but would not be expected to cause impairment of renal concentration or hypernatremia.
2019.58
A 40-year-old female sees you because of burning upper abdominal and chest pain and an acidic taste in her mouth after nearly every meal. She has pain at night that sometimes keeps her awake, but she does not have any nausea, vomiting, difficulty swallowing, bloating, bloody stools, or weight loss. She does not smoke.
Which one of the following would be the most appropriate next step?
A) Test for Helicobacter pylori and treat if present
B) Start a 4- to 8-week trial of a proton pump inhibitor
C) Order abdominal ultrasonography
D) Schedule esophagogastroduodenoscopy
E) Refer to a surgeon to consider fundoplication
B) Start a 4- to 8-week trial of a proton pump inhibitor (REVIEW: 2018.183 )
Patients with symptoms typical for GERD can be treated conservatively initially unless there are warning signs such as anemia, weight loss, evidence of bleeding or obstruction, dysphagia, or persistent symptoms despite maximal treatment, or the patient is age 50 or over. In the absence of any of these concerns, medical therapy with a proton pump inhibitor can be initiated. While H2 histamine blockers can also treat reflux symptoms they are somewhat less effective, and stepwise therapy may increase costs.
Routine testing for Helicobacter pylori in patients with GERD alone is not recommended because treating H. pylori has been shown in some studies to increase esophagitis and GERD symptoms. However, in the presence of dyspepsia (fullness, bloating, nausea), which can be associated with GERD, testing for and treating H. pylori is expected to be beneficial. This patient has classic signs and symptoms of GERD and abdominal ultrasonography would not be likely to reveal any helpful findings. In the presence of warning signs, esophagogastroduodenoscopy would be indicated to evaluate for a more serious pathology. Surgical intervention for GERD should be reserved for patients who fail maximal medical therapy or patients who are unable to take proton pump inhibitors.
2019.59
A 36-year-old male went skiing last year for the first time and when he made it to the top of the mountain he developed a headache, nausea, and dizziness, but no respiratory difficulty. That night he had difficulty sleeping. He asks for your recommendation on preventing a recurrence of the problem when he goes skiing again this year.
Which one of the following medications would you recommend he start the day before his ascent and continue until his descent is complete?
A) Acetazolamide (Diamox Sequels)
B) Aspirin
C) Dexamethasone (Decadron)
D) Tadalafil (Adcirca)
E) Zolpidem (Ambien)
A) Acetazolamide (Diamox Sequels)
Acetazolamide is the preferred agent for preventing acute mountain sickness (AMS). Multiple trials have demonstrated its efficacy in preventing AMS. Dexamethasone is a first-line treatment for acute mountain sickness of any severity but is a second-line drug for prevention because of its side-effect profile.
- Tadalafil is advised as a second-line treatment after nifedipine for the prevention and treatment of high-altitude pulmonary edema.
- Zolpidem may help with sleep but not AMS, and
- Aspirin is not recommended for prevention of AMS.
2019.60
A 25-year-old female has a heart murmur on her postpartum visit. This was first noted at the age of 20. She has been asymptomatic. The murmur is systolic and increases in intensity with Valsalva maneuvers. Further questioning reveals that her two sisters died suddenly from cardiac problems in their early twenties.
This patient should be evaluated for
A) dilated cardiomyopathy
B) hypertrophic cardiomyopathy
C) peripartum cardiomyopathy
D) restrictive cardiomyopathy
B) hypertrophic cardiomyopathy
Hypertrophic cardiomyopathy is the most common type of cardiomyopathy, with a prevalence of 1:500. It involves left ventricular hypertrophy without chamber dilatation. It is caused by autosomal dominant genetic mutations and is associated with sudden death.
- Dilated cardiomyopathy is a leading cause of heart failure but most patients are symptomatic.
- Peripartum cardiomyopathy may occur during and after pregnancy and presents as heart failure.
- Restrictive cardiomyopathy presents with right-sided heart failure.
2019.61
A 5-year-old female is brought to your office with a progressive rash on her legs (shown below) and buttocks. No rash is noted above the level of the mid-torso. Her mother also reports that the child had two episodes of bloody diarrhea 3 days ago. She also has abdominal pain and on examination she has abdominal tenderness with no rigidity but some voluntary guarding. You also note swelling and tenderness in her left wrist and right knee. A CBC, platelets, prothrombin time, and partial thromboplastin time are normal. A urinalysis reveals mild proteinuria and 5–10 RBCs/hpf.
Which one of the following is the most likely diagnosis?
A) Erythema infectiosum (fifth disease)
B) Gianotti-Crosti syndrome
C) Hemolytic uremic syndrome
D) Henoch-Schönlein purpura
E) Thrombotic thrombocytopenic purpura
D) Henoch-Schönlein purpura
Henoch-Schönlein purpura (HSP) is an IgA vasculitis that is usually diagnosed clinically. It presents as palpable purpura of the lower extremities without thrombocytopenia or coagulopathy. It is often associated with
- arthralgias and arthritis,
- abdominal pain, and
- renal dysfunction.
It is self-limited and treatment is supportive only.
- Erythema infectiosum (fifth disease) can be identified by an erythematous rash on the cheeks and a lacy reticular rash on the extremities.
- Gianotti-Crosti syndrome is a sudden papular or papulovesicular eruption on the extensor surfaces of the arms, legs, buttocks, and face, and it is not purpuric.
- Hemolytic uremic syndrome (HUS) presents with the classic triad of
- hemolytic anemia,
- thrombocytopenia, and
- kidney injury.
- Thrombotic thrombocytopenic purpura (TTP) is rare in the pediatric age group.
2019.62
A 52-year-old female sees you because of concerns about developing lung cancer. She reports that she quit smoking last month after learning that her father has stage IV lung cancer. She had smoked a pack of cigarettes per day since she was 18 years old. She has no history of cough, shortness of breath, or weight loss. She is worried about developing lung cancer and wants to know how to “catch it early.”
Based on the recommendations of the U.S. Preventive Services Task Force, in addition to providing ongoing smoking cessation support, which one of the following should you recommend?
A) A chest radiograph today
B) Low-dose chest CT today
C) Low-dose chest CT at age 55
D) No imaging, since she has already quit smoking
E) No imaging, since she is female
C) Low-dose chest CT at age 55
The 2013 U.S. Preventive Services Task Force lung cancer screening guidelines recommend annual low-dose CT screening for all adults between the ages of 55 and 80 who have a >30-pack-yearsmoking history and either currently smoke or have smoked within the past15 years (B recommendation).
2019.63
A 46-year-old female with a history of hyperthyroidism controlled with methimazole (Tapazole), 10 mg daily, returns to your office after an absence of several years. She has new symptoms of palpitations, heat intolerance, and hoarseness. A physical examination reveals an enlarged thyroid and a radioactive iodine uptake scan shows accumulation of tracer in multiple areas.
Which one of the following is the appropriate definitive treatment for this patient?
A) Methimazole alone, 20 mg daily
B) Methimazole, 20 mg daily, plus propranolol, 80 mg twice daily
C) Propylthiouracil alone, 50 mg 3 times daily
D) Radioactive iodine
E) Thyroidectomy
E) Thyroidectomy (REVIEW: 2018.208 )
This patient has a medical history, physical examination, and radioactive iodine uptake scan consistent with toxic multinodular goiter, which is the second most common cause of hyperthyroidism in the United States.
Although the addition of propranolol and an increase in methimazole may control her palpitations and other symptoms of hyperthyroidism, these measures will not permanently eliminate the problem.
Radioactive iodine ablation and thyroidectomy with subsequent thyroid hormone replacement are both appropriate treatments for toxic multinodular goiter, but thyroidectomy is indicated for this patient because she has compressive symptoms from the goiter itself.
2019.64
Which one of the following groups has the highest prevalence of syphilis?
A) Baby boomers
B) Incarcerated females
C) Men who have sex with men
D) Rural men 20–29 years of age
E) Individuals with a history of illicit intravenous drug use
C) Men who have sex with men
Factors associated with increased prevalence rates for syphilis in the United States include a history of incarceration or commercial sex work, living in the southern or western United States, residing in a major metropolitan area, African-American ethnicity, and being a male younger than 29 years of age. The risk for syphilis infection is highest among men who have sex with men and among persons who are HIV-positive.
2019.65
Which one of the following is the counseling strategy in the Stages of Change Model?
A) Quickly establishing rapport with a patient to improve compliance with recommendations for change
B) Assessing the patient’s motivation for change and determining where they are in the process
C) Focusing on a specific aspect of a problem and offering strategies for coping
D) Providing education regarding the behavior in which a change is recommended
E) Providing direct advice regarding steps for making a lifestyle change
B) Assessing the patient’s motivation for change and determining where they are in the process
The Stages of Change Model assesses the patient’s motivation for change and determines which stage of the change process the patient is in. The stages include
- Precontemplation,
- Contemplation,
- Preparation,
- Action, and
- Maintenance.
Understanding this helps guide counseling strategies for each individual patient.
2019.66
A 42-year-old female presents with a 2-week history of throbbing medial heel pain that is most painful when she first steps out of bed in the morning. The pain improves after she walks around for several minutes.
Which one of the following is the most likely diagnosis?
A) Achilles tendinopathy
B) Calcaneal stress fracture
C) Neuroma
D) Plantar fasciitis
D) Plantar fasciitis (REVIEW: 2018.170 )
Plantar fasciitis is the most common cause of heel pain, affecting more than 2 million people each year. The pain is typically worst when the patient first gets out of bed and improves with activity.
- Calcaneal stress fractures follow an increase in activity, and the pain tends to worsen with activity and is eventually present all of the time.
- Achilles tendinopathy is an aching pain that also worsens with increased activity, and there is often tenderness along the tendon.
- Neuromas present with a burning, tingling, or numb sensation and a painful lump.
2019.67
A 55-year-old patient with a history of alcoholism is admitted through the emergency department with acute pancreatitis. Which one of the following tests performed at the time of admission can best predict the severity of pancreatitis?
A) Hematocrit
B) C-reactive protein
C) Serum amylase
D) Serum lipase
E) CT of the abdomen
A) Hematocrit
Knowing the severity of pancreatitis helps predict how aggressive management should be. Hematocrit, BUN, and creatinine levels are the most useful predictors of the severity of pancreatitis, reflecting the degree of intravascular volume depletion.
- C-reactive protein is often elevated, but it is not as useful as hematocrit for predicting severity.
- Serum amylase and lipase have no prognostic value.
- CT evidence of severe pancreatitis lags behind clinical and laboratory evidence, and early CT underestimates the severity of the acute process.
2019.68
According to state child abuse mandatory reporter laws, which one of the following is the threshold that must be met for physicians to make a report to local child protective services or law enforcement?
A) If they cannot prove that abuse or neglect did not occur
B) If they consider the possibility of abuse or neglect, even if further evaluation indicates that it is unlikely
C) If they observe signs or symptoms that may occur in cases of abuse or neglect, but that are nonspecific for abuse or neglect
D) If they suspect that a child has experienced abuse or neglect
E) Only if they have proof that abuse or neglect has occurred
D) If they suspect that a child has experienced abuse or neglect
Although each state has its own laws regarding obligations to report child abuse, all 50 U.S. states require physicians, whether as a specified professional group or as a part of universal mandated reporting, to report a suspicion of child abuse. The standard is generally suspicion or cause to believe that abuse has occurred.
If the possibility of abuse is briefly considered but rejected, or if nonspecific signs are present that do not create a significant suspicion of abuse, this standard is not met.
There is no burden of proof placed on the physician to make such a report.
2019.69
A 34-year-old male with sickle cell disease has a new onset of mild to moderate thirst and polyuria. He ate a large meal about 2 hours ago.
An examination reveals a BMI of 32 kg/m2. Results of a urinalysis performed by your staff include 3+ glucose and no ketones. His blood glucose level is 288 mg/dL and his hemoglobin A1c is 5.2%.
Which one of the following would be most appropriate at this point to help diagnose and monitor this patient’s glycemic control?
A) A serum fructosamine level
B) A repeat hemoglobin A1c
C) A 2-hour glucose tolerance test
D) Hemoglobin electrophoresis
E) Referral to an endocrinologist
A) A serum fructosamine level
This patient with sickle cell disease has a new onset of diabetes mellitus. Hemoglobinopathies falsely lower hemoglobin A1c as a result of hemolysis and abnormal glycation.
- Fructosamine correlates well with hemoglobin A1c levels and is recommended instead of hemoglobin A1c for monitoring glucose control in patients with diabetes and hemoglobinopathies.
A 2-hour glucose tolerance test or hemoglobin electrophoresis would not provide useful information.
Referral to an endocrinologist is not indicated at this point because the patient has not failed primary care management.
2019.70
A 35-year-old male presents with a 2-week history of lower extremity edema. He is in good health and does not take any medications. You note weight gain, and mild dyspnea with exertion. An examination is unremarkable except for 2+ to 3+ pitting edema of the lower extremities to his knees bilaterally. A CBC and metabolic panel are unremarkable except for a low albumin level. A urinalysis reveals 3+ protein on the dipstick with no microscopic findings.
Which one of the following would be the most appropriate next step?
A) Urine microscopy to check for eosinophils
B) A spot urine protein/creatinine ratio
C) Renal ultrasonography
D) Echocardiography
E) Referral for a renal biopsy
B) A spot urine protein/creatinine ratio (REVIEW: 2018.55)
Individuals with nephrotic syndrome often present with edema and fatigue with no evidence of severe liver disease or heart failure. Hallmarks of this problem include heavy proteinuria, hypoalbuminemia, and peripheral edema, often with hyperlipidemia as well.
While most of these cases are idiopathic, secondary causes such as diabetes mellitus, systemic lupus erythematosus, and medication reactions should be considered.
To confirm proteinuria in the nephrotic range a spot urine protein/creatinine ratio is now suggested instead of a 24-hour collection of urine.
- Checking urine for eosinophils has been recommended in the past for evaluation for acute interstitial nephritis but subsequent studies have shown a lack of specificity and sensitivity.
- Renal ultrasonography would be indicated if the glomerular filtration rate were reduced.
- Echocardiography would be appropriate if heart failure were suspected.
- While a renal biopsy is often recommended, it is most useful in patients with suspected underlying systemic lupus erythematosus or similar disorders when a biopsy can guide management decisions and prognosis.
2019.71
A 69-year-old female with hypertension, hyperlipidemia, and coronary artery disease had a myocardial infarction 1 year ago that was treated with percutaneous stenting. She was recently diagnosed with atrial fibrillation and takes diltiazem (Cardizem) for rate control. She is also taking lisinopril (Prinivil, Zestril), atorvastatin (Lipitor), atenolol (Tenormin), and aspirin, 81 mg.
Of the following, which option would be best for thromboembolism prevention in this patient?
A) Continue the current medication regimen
B) Increase the aspirin dosage to 325 mg daily
C) Discontinue aspirin and begin apixaban (Eliquis)
D) Add clopidogrel (Plavix)
C) Discontinue aspirin and begin apixaban (Eliquis)
CHA2DS2-VASc Score
Congestive heart failure
Hypertension
Age ( > 65 = 1 point, > 75 = 2 points)
Diabetes
Stroke/transient ischemic attack, previous (2 points)
Vascular disease (peripheral arterial disease, previous myocardial infarction, aortic atheroma), and
Sex (female gender)
This patient has a CHA2DS2-VASc score of 3 (hypertension, age 65–74, female), which classifies her as high risk for thromboembolism.
Oral anticoagulation is indicated for patients with a score of 2 or more, who are at high risk for pulmonary embolism/deep vein thrombosis (PE/DVT) (SOR C). Patients with a score of 0–1 have a low to medium risk and may use aspirin with or without clopidogrel.
In patients with atrial fibrillation and stable coronary artery disease, novel oral anticoagulants are preferred (SOR A). They reduce the risk of reinfarction, stroke, and overall mortality in patients with a past history of myocardial infarction, and also help prevent PE/DVT.
2019.72
A 4-year-old male is brought to your office by his adoptive parents who are concerned about his poor behavior and intellectual development. The patient is in the 7th percentile for height and weight. There have been some indications of alcohol abuse by his biological mother.
Which one of the following facial dysmorphologies would be most consistent with a diagnosis of fetal alcohol syndrome?
A) A central chin “dimple”
B) Low-set ears
C) A smooth philtrum
D) A widened vermilion border of the lower lip
E) Elongated palpebral fissures
C) A smooth philtrum (REVIEW: 2018.21)
The classic facial dysmorphologies associated with fetal alcohol syndrome are a
- Smooth philtrum,
- Shortened palpebral fissures, and a
- Thin vermilion border of the upper lip.
Two out of these three characteristics are required for the diagnosis of fetal alcohol syndrome.
Low-set ears and a central chin dimple are not associated findings.
2019.73
A 7-year-old female is brought to your office by her mother, who says that the child has developed underarm odor and is beginning to develop acne. The patient has an unremarkable history, was born at full term, and has no chronic medical problems. There is no family history of endocrine disorders or precocious puberty.
An examination reveals normal vital signs and a normal BMI, and her height is stable on the growth curve with no increased velocity. Her sexual maturity rating is stage 1. She has some open and closed comedones on her forehead and back.
Which one of the following would be most appropriate at this point?
A) Reassurance and surveillance over the next 3–6 months
B) TSH, LH, and FSH levels
C) Bone age radiography
D) Bone age radiography and TSH, LH, and FSH levels
A) Reassurance and surveillance over the next 3–6 months
Premature adrenarche without development of secondary sex characteristics is usually idiopathic and does not lead to an abnormal pattern of development.
Reassurance and surveillance over the next 3–6 months would be most appropriate at this time.
Laboratory studies and radiography warrant consideration if the patient develops secondary sex characteristics before the age of 8, or if her height velocity increases rapidly during the surveillance period.
2019.74
An otherwise asymptomatic 7-year-old male has a blood pressure above the 95th percentile for gender, age, and height on serial measurements. Which one of the following studies would be most appropriate at this time?
A) Renin and aldosterone levels
B) 24-hour urinary fractionated metanephrines and normetanephrines
C) Renal ultrasonography
D) Doppler ultrasonography of the renal arteries
E) A sleep study
C) Renal ultrasonography
Renal parenchymal diseases such as glomerulonephritis, congenital abnormalities, and reflux nephropathy are the most common cause of hypertension in preadolescent children. Preadolescent children with hypertension should be evaluated for possible secondary causes and renal ultrasonography should be the first choice of imaging in this age group.
- Renin and aldosterone levels are indicated if there is a reason to suspect primary hyperaldosteronism, such as unexplained hypokalemia.
Measurement of 24-hour urinary fractionated metanephrines and normetanephrines is used to diagnose pheochromocytomas, which are rare and usually present with a triad of symptoms including headache, palpitations, and sweating.
- Doppler ultrasonography of the renal arteries is useful for diagnosing renal artery stenosis, which should be suspected in patients with coronary or peripheral atherosclerosis or young adults, especially women 19–39 years of age, who are more at risk for renal artery stenosis due to fibromuscular dysplasia.
Sleep studies are indicated in patients who are obese or have signs or symptoms of obstructive sleep apnea.
2019.75
A 58-year-old female presents to your office after being seen in the emergency department last weekend for her first episode of renal colic. After undergoing CT urography she passed a calcium phosphate kidney stone.
Which one of the following medications in her current regimen places her at higher risk for kidney stone formation?
A) Escitalopram (Lexapro)
B) Levothyroxine (Synthroid)
C) Lisinopril (Prinivil, Zestril)
D) Metformin (Glucophage)
E) Topiramate (Topamax)
E) Topiramate (Topamax)
Topiramate (anticonvulsant or antiepileptic drug) increases the risk of kidney stones.
It is a carbonic anhydrase inhibitor, which induces a metabolic acidosis that leads to hypercalciuria and the formation of calcium phosphate stones.
The risk of kidney stones is not increased by escitalopram, levothyroxine, lisinopril, or metformin.
2019.76
A 30-year-old female presents with pain over the proximal fifth metatarsal after twisting her ankle. Radiographs reveal a nondisplaced tuberosity avulsion fracture of the fifth metatarsal.
Which one of the following would be the most appropriate initial management?
A) A short leg walking boot
B) A compressive dressing with weight bearing and range-of-motion exercises as tolerated
C) A posterior splint with no weight bearing, and follow-up in 3–5 days
D) A short leg cast with no weight bearing
E) Surgical fixation
B) A compressive dressing with weight bearing and range-of-motion exercises as tolerated
The fifth metatarsal has the least cortical thickness of all of the metatarsals. There are strong ligaments and capsular attachments on the proximal fifth metatarsal that can put significant stress on this area of the bone, leading to fractures.
Nondisplaced tuberosity fractures can generally be treated with compressive dressings such as an Aircast or Ace bandage, with weight bearing and range-of-motion exercises as tolerated.
Minimally displaced (<3 mm) avulsion fractures of the fifth metatarsal tuberosity can be treated with a short leg walking boot.
If the displacement is >3 mm, an orthopedic referral is warranted.
2019.77
A 48-year-old male presents with psychogenic erectile dysfunction in conjunction with depression. You decide that treatment of his underlying mood disorder is the best initial step.
Which one of the following antidepressants would be most appropriate?
A) Bupropion (Wellbutrin)
B) Citalopram (Celexa)
C) Nortriptyline (Pamelor)
D) Sertraline (Zoloft)
E) Venlafaxine (Effexor XR)
A) Bupropion (Wellbutrin)
When psychogenic erectile dysfunction (ED) coexists with depression, treatment of the underlying mood disorder is often an appropriate first step (SOR C).
An antidepressant that is LESS likely to worsen the ED, such as bupropion, mirtazapine, or fluvoxamine, should be chosen.
- Antidepressants that are more likely to cause sexual side effects should be avoided, including SSRIs, SNRIs, and tricyclic and tetracyclic antidepressants.
- “Effective For Sadness, Panics, & Compulsions.” (SSRI’s menomic)
- Citalopram (Celexa)
- Escitalopram (Lexapro)
- Fluvoxamine (Luvox)
- Paroxetine (Paxil)
- Sertraline (Zoloft)
Phosphodiesterase-5 inhibitors are the first line of treatment for ED (SOR A) and can be used effectively in men with depression, in combination with treatments for mood disorders.
2019.78
A 14-year-old male sees you for a well child examination. He had one dose of HPV vaccine at his last well child examination 1 year ago.
Which one of the following is true regarding HPV vaccine for this patient?
A) He does not require additional HPV vaccine
B) He should receive one dose of the vaccine now and no additional HPV vaccine in the future
C) He should receive the vaccine now and again in 4 months
D) He should receive the vaccine now and again in 6 months
E) He should receive the vaccine now, in 2 months, and in 4 months
B) He should receive one dose of the vaccine now and no additional HPV vaccine in the future
HPV vaccine is currently recommended for males and females at age 11 (first dose up to 15).
Catch-up vaccination is recommended until age
- 21 in males, and
- 26 in females.
Children who receive the first dose of the vaccine before the age of 15 and receive two doses are considered adequately vaccinated.
- If the first dose is given after age 15, a three-dose series is recommended.
2019.79
Which one of the following is an indication for considering atrial ventricular nodal ablation in a patient with atrial fibrillation?
A) Hemodynamic instability
B) Successful cardioversion to sinus rhythm
C) Inability of the patient to be anticoagulated
D) Atrial fibrillation refractory to medical therapy
E) Persistent atrial fibrillation in a patient who has been successfully rate controlled and anticoagulated for several years
D) Atrial fibrillation refractory to medical therapy
Atrial ventricular nodal ablation is recommended for patients whose atrial fibrillation is refractory to medical therapy, and requires that patients be anticoagulated for at least 1 month prior to the procedure and for several months afterward (SOR C).
- Patients with atrial fibrillation who are hemodynamically unstable should be considered for emergent cardioversion (SOR C).
Atrial ventricular nodal ablation is not necessary for patients successfully converted to sinus rhythm or for those who are successfully treated with medical interventions for rate control and anticoagulation.
2019.80
An otherwise healthy 42-year-old male presents to your office with low back pain that started a week ago after he lifted a heavy box. Since the time of his injury he has been having consistent pain, numbness, and tingling that radiates down the back of his right leg to his calf.
Which one of the following would you order at this time?
A) No imaging
B) Plain radiography
C) CT
D) MRI
A) No imaging
Uncomplicated acute low back pain and/or radiculopathy is a benign, self-limited condition and early imaging is associated with worse overall outcomes and is likely to identify minor abnormalities even in asymptomatic patients.
- Imaging for acute low back pain should be reserved for cases that are suspicious for cauda equina syndrome, malignancy, fracture, or infection.
- In the absence of red flags such as progressive motor or sensory loss, new urinary retention or overflow incontinence, a history of cancer, a recent invasive spinal procedure, or significant trauma relative to age, imaging is not warranted regardless of whether radiculopathy is present, unless symptoms persist despite a trial of at least 6 weeks of medical management and physical therapy.